Đến nội dung

phuocdinh1999

phuocdinh1999

Đăng ký: 10-11-2013
Offline Đăng nhập: 05-11-2021 - 12:57
****-

#617019 $x_{n+1}=\frac{x_{n}-2+\sqrt{x_...

Gửi bởi phuocdinh1999 trong 26-02-2016 - 15:27

Cho dãy số $(x_{n})$ xác định bởi $x_{1}=2,1$ và $x_{n+1}=\frac{x_{n}-2+\sqrt{x_{n}^{2}+8x_{n}-4}}{2}.$
Đặt $y_{n}=\sum_{i=1}^{n}\frac{1}{x_{i+1}^{2}-4}.$ Tìm $\lim_{n \mapsto +\infty}y_{n}.$

+ CM: $\lim_{n \rightarrow +\infty}x_n=+ \infty$

+ $x_{n+1}=\frac{x_{n}-2+\sqrt{x_{n}^{2}+8x_{n}-4}}{2}$

Chuyển vế, bình phương được $x_{n+1}^2-x_nx_{n+1}+2x_{n+1}-3x_n+2=0 $

$\Rightarrow \frac{1}{x_n^2-4}=\frac{1}{x_n-2}-\frac{1}{x_{n+1}-2}$

$\Rightarrow y_n=\frac{1}{x_1-2}-\frac{1}{x_{n+1}-2}$

Do $\lim_{n \rightarrow +\infty}x_n=+ \infty$ nên $\lim y_n=\frac{1}{0,1}=10$




#589651 Đăng ký tham gia dự thi VMEO IV

Gửi bởi phuocdinh1999 trong 18-09-2015 - 19:18

Họ tên: Lê Phước Định

Nick trong diễn đàn: phuocdinh1999

Năm sinh: 1999

Hòm thư: [email protected]

Dự thi cấp: THPT




#558728 ĐỀ THI OLYMPIC CHUYÊN KHOA HỌC TỰ NHIÊN NĂM 2015

Gửi bởi phuocdinh1999 trong 11-05-2015 - 13:07

 

Ngày thi thứ hai

 

Câu VI. Cho $x,y,z>0$ và $xy+yz+xz=1$. CMR

$\frac{x}{\sqrt{yz}+\sqrt{3}}+\frac{y}{\sqrt{xz}+\sqrt{3}}+\frac{z}{\sqrt{xy}+\sqrt{3}}\leq \frac{1}{4\sqrt{3}xyz}$

 

---------------------------------------------------------------------------

 

 

Câu VI:

Đổi biến $(a,b,c)=(\sqrt{xy};\sqrt{yz};\sqrt{xz}). $

Từ GT suy ra $a^2+b^2+c^2=1$. BĐT trở thành: $\sum \frac{bc}{a^2+\sqrt{3}a}\leq \frac{1}{4\sqrt{3}abc}$

 

Ta có: $VT \leq \sum \frac{bc}{a^2+a(a+b+c)}$ (do $a+b+c\leq \sqrt{3}$) $=\sum \frac{bc}{a(a+b)+a(a+c)}$

         $\leq \frac{1}{4}\sum \frac{bc}{a(a+b)}$+$\frac{bc}{a(a+c)}=\sum \frac{c(a^2+b^2)}{4ab(a+b)}$ 

 

Ta cần CM: $\sum \frac{c^2(a^2+b^2)}{a+b} \leq \frac{1}{\sqrt{3}}$ $\Leftrightarrow \sum (c^2(a+b)-\frac{2abc^2}{a+b} )\leq \frac{1}{\sqrt{3}}$

 

$\Leftrightarrow (ab+bc+ca)(a+b+c)-3abc-2abc(\sum \frac{a}{b+c})\leq \frac{1}{\sqrt{3}}$

 

Do $(ab+bc+ca)(a+b+c) \leq \sqrt{3}$ và $\sum \frac{a}{b+c}\geq \frac{(a+b+c)^2}{2(ab+bc+ca)}$ nên ta chỉ cần CM:$\frac{1}{ab+bc+ca}+5 \geq \frac{2}{\sqrt{3}abc}$

 

Hay $\frac{2}{\sqrt{3}}\left ( \frac{1}{a}+\frac{1}{b}+\frac{1}{c} \right )\geq 1+5(ab+bc+ca)$

BĐT này đúng vì ta CM được $\frac{1}{a}+\frac{1}{b}+\frac{1}{c}\geq 3\sqrt{3}$ và $ab+bc+ca \leq 1$.




#550134 Tìm số người có mặt tại trại hè

Gửi bởi phuocdinh1999 trong 29-03-2015 - 18:05

Tại một trại hè có $k$ người. Cứ mỗi $m$ người trong trại hè ($m\geq 3$) thì có đúng 1 bạn chung. Quy ước rằng nếu $A$ là bạn $B$ thì $B$ là bạn $A$, và $A$ không là bạn của chính anh ta. Hỏi trại hè có bao nhiêu người?

Rõ ràng luôn tồn tại 2 người quen nhau là $A_1$ và $A_2$

Với 2 người trên thì tồn tại $A_3$ quen cả hai. (gt)

Với 3 người $A_1,A_2,A_3$ thì tồn tại người A_4 quen cả ba (do $m \geq 3$)

....

Làm liên tục như trên, ta được $m+1$ người $A_1,A_2,.. , A_{m+1}$ đôi một quen nhau.

Giả sử tồn tại B không thuộc nhóm trên 

 

TH1: B có ít nhất 2 người bạn, giả sử $A_1,A_2$ 

Khi đó xét $m$ người $B,A_3,A_4,...,A_{m+1}$ thì $m$ người này có ít nhất 2 bạn chung là $A_1,A_2$ (vô lý)

 

TH2: B có đúng 1 người bạn, giả sử là C

Xét $m$ người $B,C,A_4,A_5,...,A_{m+1}$ thì họ có đúng một người bạn chung là D 

Khi đó B có ít nhất 2 người bạn (mâu thuẫn)

 

Như vậy không tồn tại ai ngoài nhóm $m+1$ người trên. Vậy trại hè có $m+1$ người 




#532009 Chứng minh $\sqrt{a^2-a+1}+\sqrt{b^2-b+1}+...

Gửi bởi phuocdinh1999 trong 05-11-2014 - 20:16

Cho các số a,b,c thỏa mãn $abc=1$ Chứng minh $\sqrt{a^2-a+1}+\sqrt{b^2-b+1}+\sqrt{c^2-c+1}\geq a+b+c$

Tự hào là thành viên VMF

Theo nguyên tắc Dirichlet, trong 3 số $(a-1),(b-1),(c-1)$, tồn tại 2 số có tích không âm, giả sử $(b-1)(c-1) \geq 0$ hay $bc \geq b+c-1(*)$

 

Áp dụng Minkowski: $\sqrt{b^2-b+1}+\sqrt{c^2-c+1}=\sqrt{(b-\frac{1}{2})^2+\frac{3}{4}}+\sqrt{(c-\frac{1}{2})^2+\frac{3}{4}}\geq \sqrt{(b+c-1)^2+3}$ 

 

Xét $\sqrt{(b+c-1)^2+3}-(b+c)=\frac{4-2(b+c)}{\sqrt{(b+c-1)^2+3}+(b+c)}\geq \frac{2-bc}{\sqrt{b^2c^2+3}+bc+1}$ (theo$(*)$)

      $=\frac{2a-2}{\sqrt{3a^2+1}+a+1}$

 

Ta cần cm: $\sqrt{a^2-a+1}+\frac{2a-2}{\sqrt{3a^2+1}+a+1}\geq a$ $\Leftrightarrow ...\Leftrightarrow a^2(a-1)^2\geq 0$ (đúng)




#529983 Đề thi chọn HSG lớp 12 tỉnh Bình Định năm học 2014-2015

Gửi bởi phuocdinh1999 trong 22-10-2014 - 16:04

  Bài 2: (4 điểm)

a)      Cho p là một số nguyên tố, k là một số nguyên dương. Một đường trong được chia thành p cung bằng nhau. Tiến hành tô các cung bằng k màu khác nhau ( mỗi cung được tô bằng một màu). Hai cách tô màu được coi là giống nhau nếu cách tô này sẽ thu được từ cách tô kia qua một phép quay với tâm là tâm của đường tròn. Hỏi có bao nhiêu cách tô màu khác nhau? (Cách chứng minh định lí Fermat nhỏ bằng tổ hợp)

b)      Tìm tất cả các đa thức  thỏa mãn điều kiện: $P\left( x \right)=\sqrt{P\left( {{x}^{2}}+1 \right)-7}+6,\forall x\ge 0,P\left( 0 \right)=6$

 

b) Xét dãy số $(a_n):$ $a_0=6$ ; $a_(n+1)=a_n^2+1$

Dễ cm dãy trên tăng không bị chặn $(1)$

- Xét $Q(x)=P(x)-(x+6)$ , từ đó: $Q(x)+x=\sqrt{Q(x^2+1)+x^2}\Rightarrow Q(a_n)+a_n=\sqrt{Q(a_n^2+1)+a_n^2}$

Hay $Q(a_n)+a_n=\sqrt{Q(a_(n+1)+a_n^2}$

Ta có $Q(a_0)=Q(6)=0$, bằng quy nạp ta cm được $Q(a_n)=0\, \, \forall n\in \mathbb{N}*$

Kết hợp với $(1)$ suy ra $Q(x)$ có vô số nghiệm $\Rightarrow Q(x)\equiv 0\Rightarrow P(x)\equiv x+6$




#524495 MỘT BỔ ĐỂ BẤT ĐẲNG THỨC THÚ VỊ

Gửi bởi phuocdinh1999 trong 14-09-2014 - 17:37

                              MỘT BỔ ĐỀ BẤT ĐẲNG THỨC THÚ VỊ

                                                                                 Hoàng Đức Hưng
                                                       10 Toán - THPT Chuyên Nguyễn Trãi Hải Dương
                                                                                       14/9/2014



CM: Chuẩn hóa $a+b+c=3$. Khi đó, ta phải CM:
$$a^2b+b^2c+c^2a+abc\leq 4\qquad \qquad \qquad (*)$$
Không mất tính tổng quát, giả sử $a\geq b\geq c\geq 0$. Do đó, ta có: 
$$c(b-a)(b-c)\leq 0\Leftrightarrow b^2c-bc^2-abc+c^2a\leq 0\Leftrightarrow a^2b+b^2c+c^2a+abc\leq a^2b+b^2c+2abc$$

 

Cảm ơn bạn, bài viết rất hay :namtay. Tuy nhiên ở phần cm bổ đề thì ko ổn, vai trò các biến ko như nhau nên ko thể giả sử như vậy được! :wacko:

Để khắc phục ta chỉ cần sửa lại: $b$ là số nằm giữa $a$ và $c$ (tất nhiên cách cm ko thay đổi)

 

 

                            

$\boxed{1}$ Cho $a,b,c\geq 0$ thỏa mãn $a+b+c=3$. Khi đó, $\forall n\geq 2,n\in \mathbb{N}$ thì: 
$$a^nb+b^nc+c^na\leq 2^n$$

 

Đồng thời bạn cm kqua tổng quát này dc ko?




#522124 $\sqrt{\frac{a}{8b+c}}+\sqr...

Gửi bởi phuocdinh1999 trong 31-08-2014 - 17:49

Cho $a,b,c>0$. Chứng minh: $\sqrt{\frac{a}{8b+c}}+\sqrt{\frac{b}{8c+a}}+\sqrt{\frac{c}{8a+b}}\geq 1$




#520024 $$\sum \sqrt[3]{\frac{1}{a}...

Gửi bởi phuocdinh1999 trong 17-08-2014 - 08:55

Cho $a,b,c >0$ và $ab+bc+ca=1$. Hãy chứng minh rằng:

$$\sqrt[3]{\frac{1}{a}+6b}+\sqrt[3]{\frac{1}{b}+6c}+\sqrt[3]{\frac{1}{c}+6a }\leq\frac{1}{abc}$$

Áp dụng BĐT $(x+y+z)^3\leq 9(x^3+y^3+z^3)$, ta có:

$VT^3\leq 9\sum \left ( \frac{1}{a}+6b \right )=9\left ( \frac{ab+bc+ca}{abc}+6a+6b+6c \right )$

 

Do đó chỉ cần CM: $9a^2b^2c^2(ab+bc+ca+6abc(a+b+c))\leq 1$

 

Điều này đúng theo AM-GM: $a^2b^2c^2\leq \frac{1}{27}$ và $abc(a+b+c)\leq \frac{(ab+bc+ca)^2}{3}=\frac{1}{3}$

Đẳng thức xảy ra khi $a=b=c=1$




#519327 $x^{2002}+y^{2002}=2003^{2001}(x^{3}+y^{3})$

Gửi bởi phuocdinh1999 trong 13-08-2014 - 18:41

Giải PT nghiệm nguyên
$$x^{2002}+y^{2002}=2003^{2001}(x^{3}+y^{3})$$

Bổ đề: $a^2+b^2\vdots p$ với $p$ là SNT có dạng $4k+3$ $\Leftrightarrow a\vdots p$ và $b\vdots p$

 

Trở lại bài toán: Ta có $x^{2002}+y^{2002}\vdots 2003\Rightarrow x\vdots 2003;y\vdots 2003\Rightarrow x=2003x_1;y=2003y_1$

$PT\Leftrightarrow x_1^{2002}+y_1^{2002}=2003^2(x_1^3+y_1^3)$

Tương tự như trên: $x_1=2003x_2;y_1=2003y_2$

$PT\Leftrightarrow 2003^{1997}.(x_2^{2002}+y_2^{2002})=x_2^3+y_2^3$

$VT\geq VP\Rightarrow x_2=y_2=0\Rightarrow x=y=0$

 

Vậy $PT$ có nghiệm $(x,y)=(0,0)$




#518850 Hãy tìm tất cả các giá trị $k\geq2$ sao x,y,z thỏa mãn điều ki...

Gửi bởi phuocdinh1999 trong 10-08-2014 - 19:55

Cho ba số thực dương $x,y,z$ thỏa mãn điều kiện

$$\left\{ \begin{array}{l} xyz \leq 2 \\ \dfrac{1}{x^2}+\dfrac{1}{y^2}+\dfrac{1}{z^2} <k \end{array} \right. $$
Hãy tìm tất cả các giá trị $k\geq 2$ sao cho $x,y,z$ là ba cạnh của tam giác.

Giả sử $x$ là số lớn nhất

Cho $y=z=1,x\rightarrow 2\Rightarrow k\rightarrow \frac{9}{4}$

Ta CM $k=\frac{9}{4}$ là hằng số lớn nhất thỏa mãn, tức là CM bài toán tương đương với bài toán gốc như sau:

   

      " Cho $x,y,z>0$, $x \geq y+z$ và $xyz \leq 2$. Chứng minh $\frac{1}{x^2}+\frac{1}{y^2}+\frac{1}{z^2}\geq \frac{9}{4}$  $(*)$"

 

Ta có: $2\geq yz(y+z)\geq 2yz\sqrt{yz}\Rightarrow yz\leq 1(1)$

 

Do đó: $\frac{1}{x^2}+\frac{1}{y^2}+\frac{1}{z^2}\geq \frac{y^2z^2}{4}+\frac{2}{yz}$ (do $xyz\leq 2$ và $AM-GM$)

$\Leftrightarrow \sum \frac{1}{x^2}\geq \left ( \frac{y^2z^2}{4}+\frac{1}{4yz}+\frac{1}{4yz} \right )+\frac{3}{2yz}\geq \frac{3}{4}+\frac{3}{2}=\frac{9}{4}$ (theo $AM-GM$ và $(1)$)

 

Vì vậy $(*)$ được CM. Trở lại bài toán gốc, nếu $x,y,z$ ko phải độ dài ba cạnh tam giác thì $x \geq y+z$ và $xyz \leq 2$$\Rightarrow \sum \frac{1}{x^2}\geq \frac{9}{4}=k$ (mâu thuẫn)

 

Vậy với mọi $k$, $2\leq k<\frac{9}{4}$ thì $x,y,z$ là độ dài $3$ cạnh tam giác




#514623 Hỏi có thể phủ kín hình vuông 10x10 bằng các hình T.

Gửi bởi phuocdinh1999 trong 22-07-2014 - 17:13

Tô màu các ô trắng đen xen kẽ như trong bàn cờ vua.

Mỗi hình $T$ chiếm $3$ ô đen, $1$ ô trắng ($1$) hoặc $3$ trắng, $1$ đen($2$)

Giả sử phủ kín được, thì số hình loại $1$ bằng loại $2$ bằng $a$.

Khi đó, số ô trắng là: $3a+a=50$ (do bảng có $100$ ô)

Suy ra $a=12,5$ (vô lý) . Vậy ko thể phủ kín được bảng




#513856 Trường Hè Toán Học 2014-Đề Kiểm Tra Chất Lượng

Gửi bởi phuocdinh1999 trong 19-07-2014 - 09:51

 

 

$\boxed{\text{Bài 2}}$ Tìm hằng số k nhỏ nhất sao cho với mọi số thực dương a,b,c thỏa mãn $a+b+c=3$ thì luôn có bất đẳng thức:

$k(a^4+b^4+c^4-3)\geq a^3+b^3+c^3+3abc-6$

 

 

Cho $a=2;b=c=\frac{1}{2} \Rightarrow k\geq \frac{2}{7}$. Ta CM đây là hằng số nhỏ nhất bằng dồn biến:

Giả sử $a\geq b\geq c$

$BDT\Leftrightarrow 2(a^4+b^4+c^4)-7(a^3+b^3+c^3)-21abc+36\geq 0$

 

Đặt $f(a;b;c)=2\sum a^4-7\sum a^3-21abc+36$ thì

 

$f(a;\frac{b+c}{2};\frac{b+c}{2})=2a^4+4\left ( \frac{b+c}{2} \right )^4-7a^3-14\left ( \frac{b+c}{2} \right )^3-21a\left ( \frac{b+c}{2} \right )^2+36$

 

$f(a;b;c)-f(a;\frac{b+c}{2};\frac{b+c}{2})=\frac{1}{4}(b-c)^2(7b^2+10bc+7c^2+21a-21b-21c)\geq 0$ (do $a\geq b\geq c$)

 

Đặt $t=\frac{b+c}{2}$ thì $a=3-2t (0<t<\frac{3}{2})$

Ta có $f(a;b;c)\geq f(a;\frac{b+c}{2};\frac{b+c}{2})=...=9(t-1)^2(2t-1)^2\geq 0$

BĐT được CM, đẳng thức xảy ra khi $(a;b;c)=(1;1;1) $hoặc $(2;\frac{1}{2};\frac{1}{2})$ và các hoán vị

Vậy $Min k=\frac{2}{7}$




#510751 Đề thi khối A, A1

Gửi bởi phuocdinh1999 trong 04-07-2014 - 15:57

Lời giải của mình:

Ta có: $P=\frac{x^2}{x^2+yz+x+1}+\frac{y+z}{x+y+z+1}-\frac{1+yz}{9}$ 

$=\frac{2x^2}{2x^2+2yz+2x+x^2+y^2+z^2}+\frac{y+z}{x+1+y+z}-\frac{x^2+y^2+z^2+2yz}{18}$

$=\frac{2x^2}{3x^2+(y+z)^2+2x}+\frac{y+z}{x+1+y+z}-\frac{x^2+(y+z)^2}{18}$ 

Đặt $a=x;b=y+z$,ta có: $a^2+b^2=x^2+(y+z)^2 \geq x^2+y^2+z^2=2$

Áp dụng BĐT $Cauchy$ ta có:

$a+b=x+y+z \leq \sqrt{3(x^2+y^2+z^2)}=\sqrt{6}$ $a+b=x+y+z \geq \sqrt{x^2+y^2+z^2}=\sqrt{2}$ 

Vậy $\sqrt{2}\leq a+b \leq \sqrt{6}$\\

Áp dụng BĐT $Cauchy$,ta có: $P=\frac{2a^2}{3a^2+b^2+2a}+\frac{b}{a+b+1}-\frac{a^2+b^2}{18}$ 

$=\frac{a}{a+(\frac{b^2}{2a}+\frac{a}{2})+a}+\frac{b}{a+b+1}-$$\frac{a^2+b^2}{18}$ 

 

Cách khác ko dùng đạo hàm: (dựa trên lời giải của Math Is Love)

Ta có: $P\leq \frac{a+b}{a+b+1}-\frac{a^2+b^2}{18}=1-\frac{1}{a+b+1}-\frac{a^2+b^2}{18}$

Đặt

$Q=\frac{1}{a+b+1}+\frac{a^2+b^2}{18}\geq \frac{2}{(a^2+1)+(b^2+1)+2}+\frac{a^2+b^2}{18}=\frac{2}{t+4}+\frac{t}{18}(t=a^2+b^2\geq 2)$

$\Rightarrow Q\geq \frac{2}{t+4}+\frac{t+4}{18}-\frac{2}{9}\geq \frac{2}{3}-\frac{2}{9}=\frac{4}{9}(AM-GM)$

$\Rightarrow P\leq 1-Q\leq \frac{5}{9}$

Đẳng thức xảy ra khi $a=b=1$ hay $(x;y;z)=(1;0;1);(1;1;0)$




#510664 Chứng minh luôn chọn được $n$ hàng và $n$ cột sao cho các...

Gửi bởi phuocdinh1999 trong 04-07-2014 - 09:13

Cho hình vuông đơn vị $2n*2n$. Người ta tô màu $3n$ ô bất kỳ của hình vuông. Chứng minh luôn chọn được $n$ hàng và $n$ cột sao cho các hàng và cột đó chứa tất cả $3n$ ô đã tô.